4
$\begingroup$

If $\sum_{p\mid n} 1=\omega(n)$, with $1\leq t\leq q-1$ an integer, and $q$ an arbitrary prime.

Then what known asymptotics or bounds can be obtained for $\sum_{n\leq x}e^{2\pi i t\omega(n)/q}$

More specifically I am looking for an estimate on,

$$\sum_{p\leq x}\frac{1}{p}\sum_{n\leq x}\sum_{t=1}^{p-1}e^{2\pi i t \omega(n)/p}$$

I am aware that for a fixed constant $c$ that , $e^{c\omega(n)}$ is multiplicative if that helps at all, so that

$$\sum_{n=1}^\infty\frac{e^{2\pi i t\omega(n)/q}}{n^s}=\prod_{p}(1+\frac{e^{2\pi i t/q}}{p^s-1})$$

$\endgroup$
2
  • 1
    $\begingroup$ It seems to me that you are trying to estimate the summatory function for $\omega(\omega(n))$. $\endgroup$ Aug 16, 2013 at 1:16
  • $\begingroup$ Yes, but I would also be interested in estimates on the partial sums of $\zeta^{\omega(n)}$. $\endgroup$ Aug 16, 2013 at 1:49

2 Answers 2

4
$\begingroup$

The sum $\sum_{n\le x} e^{2\pi i t \omega(n)/q}$ is the sum of a multiplicative function whose value at a prime $p$ is $e^{2\pi i t/q}$. Asymptotics for partial sums of multiplicative functions $f$ with $f(p)=z$, a given number, are well understood by work of Selberg and Delange. The answer is $$ \sim C x (\log x)^{z-1}/\Gamma(z), $$ where $C = C(f)$ is the limiting value of $(\sum_{n=1}^{\infty} f(n)/n^s) \zeta(s)^{-z}$ as $s\to 1$, and $\Gamma(z)$ is the usual $\Gamma$ function.

See Selberg's paper Note on a paper of L.G. Sathe; or Tenenbaum's book on analytic and probabilistic number theory for more on this topic.

One can also understand the sum of $\omega(\omega(n))$ for $n\le x$ more directly. Almost all numbers up to $x$ have about $\log \log x$ prime factors (up to deviations of about $\sqrt{\log \log x}$), and then a number of size $\log \log x$ would have about $\log \log \log \log x$ prime factors. So I would guess this answer to be about $x \log \log \log \log x$.

$\endgroup$
1
  • $\begingroup$ I wouldn't be so sure about that estimate for $\omega(\omega(n))$ for example similar heuristics wouldn't work on the sum, $$\sum_{n\leq x} \omega(d(n))=x\ln(\ln(x))+O(x)$$ Also I can't seem to find the paper, could you post a link please. $\endgroup$ Aug 16, 2013 at 3:50
3
$\begingroup$

As Lucia says, in order to estimating $\sum_{n\le x} v^{\omega(n)}$ the key phrase is Selberg-Delange method (there is a chapter about it in Tenenbaum's book "Intro to analytic and probabilistic number theory").

As far as estimating $S(x) =\sum_{n\le x} \omega(\omega(n))$ is concerned, which is what you actually want to do, it is be possible to apply a more direct approach, as Lucia also indicates. I shall elaborate on this below. (Note that this approach is more direct only on the surface, as it depends on results derived via the Selberg-Delange method.) The starting point is the identity $$ S(x) = \sum_{r\ge1} \omega(r) \pi_r(x) , $$ where $$ \pi_r(x)=\#\{n\le x: \omega(n)=r\}. $$ Now, we know that \begin{align*} \pi_r(x) &=\left( \lambda\left(\frac{r-1}{\log\log x} \right) + O\left(\frac{r}{(\log\log x)^2}\right)\right)\frac{x}{\log x} \frac{(\log\log x)^{r-1}}{(r-1)!} \\ &\asymp \frac{x}{\log x} \frac{(\log\log x)^{r-1}}{(r-1)!}, \end{align*} uniformly for $r\le 10\log\log x$, where $$ \lambda(z) = \frac{1}{\Gamma(z+1)} \prod_p\left(1+\frac{z}{p-1}\right)\left(1-\frac1p\right)^{z} $$ (This is Theorem 4 in page 205 of Tenenbaum's book, 1995 english edition). Larger $r$ should contribute very little to $S(x)$. One can use the Hardy-Ramanujan theorem to bound their contribution, which says that $$ \pi_r(x) \le \frac{Ax}{\log x} \frac{(\log\log x+B)^{r-1}}{(r-1)!}, $$ for two absolute constants $A$ and $B$ (and all $r\ge1$ and $x\ge3$). So we essentially have that \begin{align*} S(x) &\sim \sum_{r\le 10\log\log x} \omega(r) \pi_r(x) \\ &\sim \sum_{r\le 10\log\log x} \omega(r) \lambda\left(\frac{r-1}{\log\log x} \right) \frac{x}{\log x} \frac{(\log\log x)^{r-1}}{(r-1)!} . \end{align*} Most of the contribution to this sum should come from integers $r=\log\log x+O(\sqrt{\log\log x})$ which have about $\log\log\log\log x$ prime factors. Also, for such $r$ we have that $\lambda((r-1)/\log\log x)\sim1$. The sum of $(\log\log x)^{r-1}/(r-1)!$ over such $r$ should be $\sim \log x$ (this is a baby version of the Central Limit Theorem). So it seems that $S(x)\sim x\log\log\log\log x$.

You might be able to obtain this result using your initial approach too, you have to be careful about the dependence of your estimates on $p$ though.

$\endgroup$

Your Answer

By clicking “Post Your Answer”, you agree to our terms of service and acknowledge you have read our privacy policy.

Not the answer you're looking for? Browse other questions tagged or ask your own question.